3

自作問題まとめてみた

206
0
$$$$

この記事は,仮の人さん主催の Advent Math Calendar の21日目になります.

12/19現在,ここまでしか書いていません.やばいです.何かテーマを決めて書きたかったのですが,記事になりそうになかったので,私の今までの作問を紹介したいと思います.

RKC004

関数$f:\mathbb{Z}\rightarrow\mathbb{Z}$であって,任意の$x,y\in\mathbb{Z}$について
$$f(f(x)+y)=x+f(y)$$を満たすものをすべて求めよ.

QMT001

正三角形$ABC$とその内部の点$P$について,以下が成立しています.
$$AP^2+BP^2+CP^2=17,AP^4+BP^4+CP^4=131$$$\triangle ABC$の面積を$S$とすると,$S^2$は互いに素な正整数$a,b$を用いて$\dfrac{b}{a}$と書けるので,$a+b$を解答してください.

QMT002

$1$から$10$までの数が書かれたカード$10$枚から$3$枚を選ぶ方法$_{10}\mathrm{C}_3$通りのうち,選んだ$3$枚が以下の条件を満たすものすべてについて,$3$枚に書かれた数のの総和を解答してください.
・選んだ$3$枚からどの$2$枚をとっても,カードの数の和は$11$でない.

作問チーム戦

数列$a_n$$a_1=\sqrt{2-2\cos{\left(\dfrac{882}{5}\right)^\circ}},a_2=1-2\cos{\left(\dfrac{882}{5}\right)^\circ}$として,以下の漸化式を満たします.
$$a_{n+1}=\dfrac{(a_n)^2-1}{a_{n-1}}(n=2,3,4,\cdots)$$このとき,$\lfloor (a_{49})^2\rfloor$の値を解答してください.ただし,$-0.998027<\cos{\left(\dfrac{882}{5}\right)^\circ}<-0.998026$を用いても構いません.

RKC005

以下を満たすための正整数$N$の条件を求めてください.
$2$つの$N$面サイコロ$A,B$であって,各面には正整数が書かれている.ただし$A$$B$の目の付き方は異なっていて,$A$$B$を振ったときの目の和の確率分布は,$1$から$N$までが$1$回ずつ書かれた$N$面サイコロを$2$回振ったときの目の和の確率分布と等しくなるようなものが存在する.ただし,立体的な目の付き方が異なるだけのサイコロは同一視する.

ΠMC002

$AB=100,AC=200$なる$\triangle ABC$において,$A$を通る類似中線と$BC$の交点を $X$とします.$BX,CX$の長さがいずれも正整数値であるとき,$AX$のとりうる正整数値としてあり得るものの総和を求めてください.

OMC不採用問題

$$(45+1)(45^2+1)\cdots(45^{512}+1)(45^{1024}+1)$$を素数$2017$で割った余りはいくつですか?

 以下,解説です.

余白

ああ
あああ
ああああ
あああああ
ああああああ
あああああああ
ああああああああ
あああああああああ
ああああああああ
あああああああ
ああああああ
あああああ
ああああ
あああ
ああ

余白

問題1 解説

$Ans.f(x)=x,-x$
この問題は,FEを作りたいな〜と思いながら適当に紙に書いてみたら解けてしまった問題です.与式への代入を$P(x,y)$で表します.
まず,$P(x,0)$とすると,$f(f(x))=x+f(0)$なので,$f$が全単射であることがわかります.$P(0,x)$より,$f(f(0)+x)=f(x)$ですが,$f$は単射なので,$f(0)+x=x\iff f(0)=0$がわかります.よって$f(f(x))=x$です.ここで,$P(f(x),y)$より,$f(x+y)=f(x)+f(y)$が得られます.何やら見たことがある式です.そう,これはコーシーの関数方程式の形をしています.これは実は$\mathbb{Q}\rightarrow\mathbb{Q}$までなら簡単に解くことができます.上式への代入を$Q(x,y)$で表します.$Q(x,1)$より$f(x+1)=f(x)+f(1)$です.$f(0)=0$なので,帰納的に$f(x)=f(1)x(x\geq0)$となります.また,$Q(x,-x)$より$0=f(x)+f(-x)$なので,$f(x)=f(1)x(x<0)$なので結局$f(x)=f(1)x$です.あとは最初の式に代入してあげることで,$f(1)^2=1$なので,$f(x)=x,-x$が解となります.これは確かに与式を満たしますね.

問題2 解説

奇妙な条件が与えられています.このままでは扱いづらいので,図形を回転してみましょう.
fig fig
 一般に,$AP^2+BP^2+CP^2=x,AP^4+BP^4+CP^4=y$ とおくと,$S$ は以下の様に表されます.以下証明です.
$$S=\dfrac{3\sqrt{x^2-2y}+\sqrt3x}{8}$$
$\triangle PAB,\triangle PBC,\triangle PCA$ をそれぞれ時計回りに $60^\circ$ 回転させてできる六角形に着目すると,
$2S=(一辺PAの正三角形)+(一辺PBの正三角形)+(一辺PCの正三角形)+3\times(三辺がPA,PB,PCの三角形)$より,
$2S=\dfrac{\sqrt3}{4}PA^2+\dfrac{\sqrt3}{4}PB^2+\dfrac{\sqrt3}{4}PC^2+3\times\dfrac{\sqrt{2(PA^2PB^2+PB^2PC^2+PC^2PA^2)-(PA^4+PB^4+PC^4)}}{4}=\dfrac{\sqrt3}{4}x+3\times\dfrac{\sqrt{x^2-2y}}{4}$
ゆえに $S=\dfrac{3\sqrt{x^2-2y}+\sqrt3x}{8}$
 よって,$S^2=\dfrac{507}{16}$ とわかるので,特に解答すべき値は $\bf{523}$です.

問題3 解説

まず,カードを$(1,10),(2,9),(3,8),(4,7),(5,6)$$5$組に分けます.そこから$3$組を選びます.選んだ$3$組からカードを選ぶ$2^3$通りにおいて,カードに書かれた数の積の総和は$11^3$なので,答えは$_5\mathrm{C}_3\times11^3=\bf{13310}$

問題4 解説

 まず,半径$1$の正$100$角形上のある$1$点を${A_1}$とし,そこから反時計回りに${A_2,A_3,\cdots,A_{50},A_{51}}$$51$個に名前をつけ,${A_1A_3}=b_1,{A_1A_4}=b_2,\cdots,{A_1A_{50}}=b_{48},{A_1A_{51}}=b_{49}$とします.ここで,${A_1,A_2},\cdots,{A_{50},A_{51}}$から任意に$4$点を選びそれらを結ぶ四角形を作ると,それは円に内接します.四角形を,${A_1A_2A_3A_4,A_1A_2A_4A_5,\cdots,A_1A_2A_{49}A_{50},A_1A_2A_{50}A_{51}}$のように選び,Ptolemyの定理を適用すると,$(b_1)^2=b_2+1,(b_2)^2=b_3b_1+1,(b_3)^2=b_4b_2+1,\cdots,(b_{50})^2=b_{51}b_{49}+1$となり,$n\geq 2$ のとき,漸化式
$$b_{n+1}=\dfrac{(b_n)^2-1}{b_{n-1}}(n=2,3,4,\cdots)$$
 を満たします.ここで,$\triangle {A_1A_2A_3}$ に余弦定理を適用すると,$(b_1)^2=2-2\cos{\left(\dfrac{882}{5}\right)^\circ}$であり,$b_2=(b_1)^2-1=1-2\cos{\left(\dfrac{882}{5}\right)^\circ}$である.つまり,数列${b_n}$は数列${a_n}$に他ならない事がわかります. よって,$a_{49}=b_{49}=\rm{A_1A_{51}}$となり,これは,この正$100$角形に外接する円の直径に相当します.円の中心を${O}$,半径を$x$とし,$\triangle{OA_1A_2}$に余弦定理を適用すると,
$$1=2x^2-2x^2\cos{\left(\dfrac{18}{5}\right)^\circ}$$
 これを解くと,$x^2=\dfrac{1}{2\left(1-\cos{\left(\dfrac{18}{5}\right)^\circ}\right)}=\dfrac{1}{4\sin^2{\left(\dfrac{9}{5}\right)^\circ}}$
 あとは,適切な評価により答えは$\mathbf{1013}$です.

問題5 解説

まず,問題の状況を数式で捉えてみましょう.母関数を使います.$N$面サイコロの母関数$h$は,$h(x)=x+x^2+\cdots+x^N$です.ここで,ある非負整数係数多項式$f,g(f\neq g)$が存在し,$f(x)g(x)=h(x)^2,f(0)=g(0)=0,f(1)=g(1)=N$を満たせば良いです.

$N$$1$または素数の時

$h$は既約なので条件を満たしません.

$N$が合成数の時

$N$$1$でも$N$でもない約数を$d$として,
$f(x)=x(1+x+\cdots+x^{d-1})(1+x+\cdots+x^{\frac{n}{d}-1})$
$g(x)=x(1+x^{\frac{n}{d}}+\cdots+x^{n-\frac{n}{d}})(1+x^d+\cdots+x^{n-d})$
とすれば条件を満たすことがわかります.

以上より,求める条件は,$N$が合成数であることです.

問題6 解説

 symmedianの基本的な性質より,以下の式が成立します.
$$\dfrac{BX}{CX}=\dfrac14$$
$BX=b(CX=4b)(20< b<60)$とおくと,Stewartの定理より,
$$AC^2\cdot BX+AB^2\cdot CX=BC(BX\cdot CX+AX^2)$$つまり,
$$AX^2=16000-4b^2=4(4000-b^2)$$であり,$4000-b^2$が平方数であることが必要です.$b$$4$の倍数であることがわかるので$b=4b’$とおくと,$64(250-b’^2)$が平方数になればよいです.探索すると$’b=9,13$が適するので,$AX=104,72$です.よって,解答すべき値は$\mathbf{176}$です.

問題7 解説

与式は$\dfrac{45^{2048}-1}{44}$と等しいです.よって,Fermatの小定理などにより,$$\dfrac{45^{2048}-1}{44}\equiv\dfrac{45^{22}-1}{44}\equiv\dfrac{(-2)^{11}-1}{44}\equiv-\dfrac12\equiv\mathbf{1013}$$となります.

 以上が自作問題のまとめになります.作問って楽しいですね〜.OMCにも問題を提出しているので,近いうちにWriterをやってみたいです!
 最後に,今回の問題を厳選するにあたり未発表の問題を発掘したので,放流したいと思います.解けた方はぜひDMまで!(JMO0.5番級くらい?)

自作問題

$f:\mathbb{R}\rightarrow\mathbb{R}$であって,任意の$x,y,z\in\mathbb{R}$に対して$$f(x+y+z)^2=f(x)^2+f(y)^2+f(z)^2+2xf(y)+2yf(z)+2zf(x)$$を満たすものをすべて求めよ.

記事は以上になります.それではまた今度!
(kkkaaaさんのツイートで知ったのですが,$2$つ以上のデバイスで編集する時は注意しないといけないみたいです.何と記事の半分が消えてしまい現在1:30です.みなさん気をつけましょうね.)

投稿日:20231220
更新日:20231221

この記事を高評価した人

高評価したユーザはいません

この記事に送られたバッジ

バッジはありません。

投稿者

OMCer(黄)です。気づいたこととか書きます。

コメント

他の人のコメント

コメントはありません。
読み込み中...
読み込み中